19
$\begingroup$

Recall that the radical of an integer $n$ is defined to be $\operatorname{rad}(n) = \prod_{p \mid n } p$.

For a paper, I need the result that $$\sum_{n \leq x} \frac{1}{\operatorname{rad}(n)} \ll_\varepsilon x^{\varepsilon} \tag{$*$},$$ for all $\varepsilon > 0$. I have a proof of this using complex analysis and Perron's formula, but this seems a bit overkill given that I'm looking for a weak upper bound for a problem in elementary number theory.

Does anyone know of a short elementary proof of the bound $(*)$? Or better yet, a reference?

$\endgroup$
1
  • 6
    $\begingroup$ The solutions given are what is sometimes called "Rankin's trick", that is, multiplying a series $n\le X$ by $(X/n)^\alpha$ and optimizing $\alpha$. My recollection is that getting an asymptotic for your sum is rather difficult (though again IIRC a log asymptotic is viable by the saddlepoint method). $\endgroup$ Nov 15, 2018 at 19:03

5 Answers 5

21
$\begingroup$

You can get away with elementary analytic number theory. Consider the series $\sum_n\frac{1}{n^{\varepsilon}\rm{rad}(n)}$. It suffices to show that it converges. However, it can be written as a product of $$ S(p)=1+p^{-1-\varepsilon}+p^{-1-2\varepsilon}+\dots=1+p^{-1-\varepsilon}\frac 1{1-p^{-\varepsilon}}\le 1+p^{-1-\frac\varepsilon 2} $$ for all but finitely many $p$. Thus $\prod_p S(p)\le C\prod_p(1+p^{-1-\frac\varepsilon 2})\le\sum_n n^{-1-\frac\varepsilon 2}<+\infty$

$\endgroup$
14
$\begingroup$

First, notice that for any squarefree $m$ and any $\varepsilon>0$ we have

notice that

$$\sum_{n:\operatorname{rad}(n)=m} \frac{1}{n^\varepsilon}=m^{-\varepsilon}\prod_{p\mid m}(1-p^{-\varepsilon})^{-1}\ll_\varepsilon d(m)/m^\varepsilon,$$

thus, the series

$$r(s)=\sum_{n=1}^{+\infty} \frac{1}{n^s\mathrm{rad}(n)}$$

converges absolutely when $\mathrm{Re}\,s>0$. Now, using multiplicativity, one has

$$r(s)=\prod_p (1+p^{-s-1}+p^{-2s-1}+\ldots)=\prod_p (1+\frac{1}{(1-p^{-s})p^{1+s}}).$$

Next, notice that for positive $\varepsilon$ we have $1-2^{-\varepsilon}\gg \varepsilon$ and $1-p^{-\varepsilon}\geq \varepsilon$ for $p>2$ and $\varepsilon<1/6$. Therefore we deduce for any $\varepsilon>0$

$$r(\varepsilon)\ll \prod_p\left(1+\frac{1}{\varepsilon p^{1+\varepsilon}}\right)\leq \zeta(1+\varepsilon)^{1/\varepsilon}.$$

As $\zeta(1+\varepsilon)=\frac{1}{\varepsilon}+O(1)$, we finally obtain

$$r(\varepsilon)\ll \varepsilon^{-1/\varepsilon}.$$

Using Rankin trick we arrive at

$$\sum_{n\leq x} \frac{1}{\mathrm{rad}(n)}\ll x^\varepsilon \varepsilon^{-1/\varepsilon}.$$

Choosing $\varepsilon=\sqrt{\frac{\ln\ln x}{2\ln x}}$ we prove that

$$\sum_{n\leq x} \frac{1}{\mathrm{rad}(n)}\leq \exp(\sqrt{(2+o(1))\ln x\ln\ln x}),$$

which is a bit non-optimal by the answer of Don. (But at least we have the correct $\ln\ln$ asymptotics)

$\endgroup$
0
11
$\begingroup$

de Bruijn studies this sum in "On the number of integers $\le x$ whose prime factors divide $n$", which was published in a 1962 volume of the Illinois J. Math.

He proves there (see Theorem 1) that $$\sum_{n \le x} \frac{1}{\mathrm{rad}(n)} = \exp((1+o(1)) \sqrt{8\log{x}/\log\log{x}}),$$ as $x\to\infty$. Of course, this implies the $O(x^{\epsilon})$ bound you were after. However, his proof (which uses a Tauberian theorem of Hardy and Ramanujan) is not as elementary as some others that have been suggested here (but gives a more precise result).

$\endgroup$
1
2
$\begingroup$

sIt seems that this argument hasn't been presented yet, so I might as well include it.

We can sort the integers $n \in [1, X]$ by their radicals, which is necessarily a square-free integer $m$. Thus we have

$$\displaystyle \sum_{n \leq X} \frac{1}{\text{rad}(n)} = \sum_{\substack{m \leq X \\ m \text{ square-free}}} \frac{1}{m} \sum_{\substack{n \leq X \\ \text{rad}(n) = m}} 1.$$

Now, $\text{rad}(n) = m$ if and only if $p | n \Rightarrow p | m$. If we write $m = p_1 \cdots p_k$, then

$$\displaystyle \sum_{\substack{n \leq X \\ \text{rad}(n) = m}} 1 = \#\{(x_1, \cdots, x_k) : x_i \in \mathbb{Z} \cap [0,\infty), p_1^{x_1} \cdots p_k^{x_k} \leq X/m\}.$$

The inequality defining the right hand side is equivalent to

$$\displaystyle x_1 \log p_1 + \cdots + x_k \log p_k \leq \log(X/m),$$

and this is just counting integer points with non-negative entries bounded by a simplex, and it is easy to see that

$$\displaystyle \# \{(x_1, \cdots, x_k) : x_1 \log p_1 + \cdots + x_k \log p_k \leq \log(X/m)\} \ll \frac{\log(X/m)}{\prod_{1 \leq i \leq k} \log(p_i)} \ll \log X.$$

EDIT: This last step is wrong, but it can be fixed. Indeed, we can arrange the $p_i$'s so that $p_1 < p_2 < \cdots < p_k$. It then follows from Davenport's lemma that

$$\displaystyle \# \{(x_1, \cdots, x_k) : x_1 \log p_1 + \cdots + x_k \log p_k \leq \log(X/m)\} = O \left(\sum_{i=0}^k \frac{(\log X/m)^{k-i}}{\prod_{1 \leq j \leq k-i} \log p_i} \right).$$

It then follows that

$$\displaystyle \sum_{n \leq X} \frac{1}{\text{rad}(n)} \ll \sum_{\substack{p_1 < \cdots < p_k \\ p_1 \cdots p_k \leq X}} \sum_{i=0}^k \frac{(\log X)^{k-i}}{\prod_{1 \leq j \leq k -i} p_i \log p_i}.$$

From here I think it is possible to get the bound $O_\epsilon(X^\epsilon)$, but it requires a somewhat more refined analysis on the interaction between the number of primes and the size of the primes.

$\endgroup$
5
  • 4
    $\begingroup$ I worry a bit about the uniformity in the "easy to see that" estimate in the $p_j$. Still, a very natural approach. $\endgroup$ Nov 16, 2018 at 0:50
  • 4
    $\begingroup$ Note that de Bruijn's estimate (quoted in my answer) shows that Greg's concern is a serious one: the sum is in fact not bounded by any fixed power of $\log{X}$. $\endgroup$ Nov 16, 2018 at 3:36
  • 2
    $\begingroup$ The volume of the simplex should involve $(\log(X/m))^k$ rather than just $\log(X/m)$. This changes the bound dramatically. $\endgroup$ Nov 16, 2018 at 13:46
  • $\begingroup$ Using a correct formula for the volume, I get $\sum_{n\le X}\frac1{\mathrm{rad}(n)}\le\prod_{p\le X}\left(1+\frac{\log X}{p\log p}\right)$, which I believe can be bounded by $\exp\left(\bigl(1+o(1)\bigr)\frac{\log X}{\log\log X}\right)$. $\endgroup$ Nov 16, 2018 at 14:58
  • $\begingroup$ Now that I see it, this might begin to explain where Gerhard Paseman got his bound. $\endgroup$ Nov 16, 2018 at 15:05
2
$\begingroup$

Here is another approach. Let $p_0$ be the largest prime with $(p_0)^{(e-1)p_0} \leq x$. The desired sum is bounded above by $P =\prod_{p}(1+\lfloor \log_p x \rfloor/p)$, where the product is over primes $p$ less than or equal to $x$.

When we pick out those terms of $P$ whose numerator is $k$, and consider the product of just those terms, we look at those primes with $p^k \lt x \leq p^{k+1}$ and the log of that product is bounded by $k$ times the sum $ S_k$ of $1/p$ over those primes. Mertens theorem gives $\log((k+1)/k)$ as an approximate value for $S_k$ for small $k$, so the subproduct is approximated by $((k+1)/k)^k$. So for $k=1$ up to just before $(e-1)p_0$, we have broken the product over larger primes than $p_0$ into sub products each bounded by $e$.

So we have an immediate upper bound on $P$ of $(1 + (\log_2 x)/2)^{\pi(p_0)}e^{(e-1)p_0}$. For $x$ not too small, this is less than $(\log x)^{\pi(p_0)}e^{(e-1)p_0}$. So far we have log of your sum is dominated by $\log P$ which in turn is dominated by $(e-1)p_0 + \pi(p_0)\log\log x$. We want this last quantity to be asymptotically less than $\epsilon\log x$.

Well, $(e-1)p_0 \leq (\log x)/(\log p_0)$, so $p_0 \lt (\log x)/f(x)$ for a function $f(x)$ which is slowly increasing. But $\pi(p_0)$ is asymptotically $( (\log x)/f(x))/(\log\log x - \log f(x))$, so the second term is only slightly bigger than $\log(x)/f(x)$, but small enough to dip below $\epsilon\log x$.

If you put in some work, you find $f(x)$ is less than but close to $\log\log x$, and far enough away for the fraction $(\log\log x)/(\log\log x - \log f(x))$ not to be a problem. Although the prime number theorem and Mertens theorem on sum 1/p are used, this should be elementary enough.

Observation 2018.11.16 Since a weak result is wanted, we can weaken some of the requirements: replace the prime number theorem by a result that bounds $\pi(p)$ from above by $Ap/\log p$ , and regroup the terms of the partial product $P$ into pieces each of which multiply to a number less than $e^2$. One should not need the full strength of Mertens for this. Or, follow the suggestion in the comment below and focus on the product of the biggest $\pi(p_0)$ terms, and show the difference between this product and the sum is sufficiently small. End Observation 2018.11.16.

Gerhard "For Some Value Of 'Enough'" Paseman, 2018.11.15.

$\endgroup$
1
  • $\begingroup$ One can also upper bound the sum by dividing it into two: one with terms where the radical includes primes bigger than p_0, and one with terms where the radical has no primes bigger than p_0. The argument above shows that the first part is less substantial than the second part. This suggests to me that looking at the second part (sum over p_0-smooth numbers) is more interesting and requires more delicacy. Gerhard "Waves Hands Over Hard Parts" Paseman, 2018.11.16. $\endgroup$ Nov 16, 2018 at 17:14

Your Answer

By clicking “Post Your Answer”, you agree to our terms of service and acknowledge you have read our privacy policy.

Not the answer you're looking for? Browse other questions tagged or ask your own question.